IMO Problem 4

Players A and B play a paintful game on the real line. Player A has a pot of paint with four units of black ink. A quantity p p of this ink suffices to blacken a (closed) real interval of length p p .In the beginning of the game, player A chooses (and announces) a positive integer N N . In every round, player A picks some positive integer m N m \leq N and provides 1 2 m \frac {1}{2^m} units of ink from the pot. Player B then picks an integer k k and blackens the interval from k 2 m \frac {k}{2^m} to k + 1 2 m \frac {k+1}{2^m} (some parts of this interval may have been blackened before). The goal of player A is to reach a situation where the pot is empty and the interval [ 0 , 1 ] [0, 1] s is not completely blackened. Decide whether there exists a strategy for player A to win in a finite number of moves.

This problem is from the IMO 2013 SLThis problem is from the IMO.This problem is part of this set .

Yes No

This section requires Javascript.
You are seeing this because something didn't load right. We suggest you, (a) try refreshing the page, (b) enabling javascript if it is disabled on your browser and, finally, (c) loading the non-javascript version of this page . We're sorry about the hassle.

0 solutions

No explanations have been posted yet. Check back later!

0 pending reports

×

Problem Loading...

Note Loading...

Set Loading...